Difference between revisions of "1958 AHSME Problems/Problem 1"

(Fixed $LaTeX$)
m (See also)
 
(One intermediate revision by one other user not shown)
Line 15: Line 15:
 
==See also==
 
==See also==
  
{{AHSME box|year=1958|before=First question|num-a=2}}
+
{{AHSME 50p box|year=1958|before=First question|num-a=2}}
 +
{{MAA Notice}}

Latest revision as of 06:10, 3 October 2014

Problem

The value of $[2 - 3(2 - 3)^{-1}]^{-1}$ is:

$\textbf{(A)}\ 5\qquad  \textbf{(B)}\ -5\qquad  \textbf{(C)}\ \frac{1}{5}\qquad  \textbf{(D)}\ -\frac{1}{5}\qquad  \textbf{(E)}\ \frac{5}{3}$

Solution

$\frac{1}{2-3(-1)}=\frac{1}{5}$, so the answer is $\boxed{\text{C}}$.

See also

1958 AHSC (ProblemsAnswer KeyResources)
Preceded by
First question
Followed by
Problem 2
1 2 3 4 5 6 7 8 9 10 11 12 13 14 15 16 17 18 19 20 21 22 23 24 25 26 27 28 29 30 31 32 33 34 35 36 37 38 39 40 41 42 43 44 45 46 47 48 49 50
All AHSME Problems and Solutions

The problems on this page are copyrighted by the Mathematical Association of America's American Mathematics Competitions. AMC logo.png